Đến nội dung

Stranger411 nội dung

Có 85 mục bởi Stranger411 (Tìm giới hạn từ 28-04-2020)



Sắp theo                Sắp xếp  

#385774 VMO 2013 - Bài 1. Hệ phương trình

Đã gửi bởi Stranger411 on 11-01-2013 - 23:00 trong Phương trình - Hệ phương trình - Bất phương trình

Cách khác: (không dùng Minkowski :D)
ĐKXĐ: $x\neq \dfrac{m\pi }{2}$, $y\neq \dfrac{n\pi }{2}$ ($m$, $n\in \mathbb{Z}$), $xy\geq 0$, $x+y\neq 0$.
Ta chứng minh:
$(\sqrt{{{\sin }^{2}}x+\frac{1}{{{\sin }^{2}}x}}+\sqrt{{{\cos }^{2}}y+\frac{1}{{{\cos }^{2}}y}})^2 + (\sqrt{{{\sin }^{2}}y+\frac{1}{{{\sin }^{2}}y}}+\sqrt{{{\cos }^{2}}x+\frac{1}{{{\cos }^{2}}x}})^2 \ge 20$

Áp dụng $\frac{1}{\sin ^2 x} + \frac{1}{\cos ^2 x} \ge 4$ $ \forall x \neq 0$, ta có:
$(\sqrt{{{\sin }^{2}}x+\frac{1}{{{\sin }^{2}}x}}+\sqrt{{{\cos }^{2}}y+\frac{1}{{{\cos }^{2}}y}})^2 + (\sqrt{{{\sin }^{2}}y+\frac{1}{{{\sin }^{2}}y}}+\sqrt{{{\cos }^{2}}x+\frac{1}{{{\cos }^{2}}x}})^2 $
$\ge 10 + 2 \sqrt{{{\sin }^{2}}x+\frac{1}{{{\sin }^{2}}x}}.\sqrt{{{\cos }^{2}}y+\frac{1}{{{\cos }^{2}}y}} + 2 \sqrt{{{\sin }^{2}}y+\frac{1}{{{\sin }^{2}}y}}.\sqrt{{{\cos }^{2}}x+\frac{1}{{{\cos }^{2}}x}}$

Vậy chỉ cần chứng minh:
$ \sqrt{{{\sin }^{2}}x+\frac{1}{{{\sin }^{2}}x}}.\sqrt{{{\cos }^{2}}y+\frac{1}{{{\cos }^{2}}y}} + \sqrt{{{\sin }^{2}}y+\frac{1}{{{\sin }^{2}}y}}.\sqrt{{{\cos }^{2}}x+\frac{1}{{{\cos }^{2}}x}} \ge 5$ $ \forall x \neq 0$
từ đó quay về chứng minh:
$ \left(\sin^2x + \dfrac{1}{\sin^2x}\right)\left(\cos^2x + \dfrac{1}{\cos^2x}\right) \geq \left(\dfrac{5}{2}\right)^2.$và $ \left(\sin^2y + \dfrac{1}{\sin^2y}\right)\left(\cos^2y + \dfrac{1}{\cos^2y}\right) \geq \left(\dfrac{5}{2}\right)^2.$

Sử dụng bất đẳng thức Cauchy-Schwarz và AM-GM ta có
$$ \begin{aligned} \left(\sin^2x + \dfrac{1}{\sin^2x}\right)\left(\cos^2x + \dfrac{1}{\cos^2x}\right) \ & \geq \left(|\sin x\cos x|+\dfrac{1}{|\sin x\cos x|}\right)^2 \\ & =\left(\dfrac{|\sin 2x|}{2}+\dfrac{1}{2|\sin 2x|}+\dfrac{3}{2|\sin 2x|}\right)^2 \\ & \geq \left( 1+\dfrac{3}{2}\right)^2=\left(\dfrac{5}{2}\right)^2.\end{aligned}$$
Ta có (đpcm).
Suy ra: $\tan x = \tan y= \pm 1$ và $x=y$
Từ đó, ta được $ x=y= \dfrac{\pi }{4}+\dfrac{k\pi }{2}$ ($k\in \mathbb{Z}$).



#400670 Tô màu các số tự nhiên

Đã gửi bởi Stranger411 on 28-02-2013 - 14:40 trong Tổ hợp và rời rạc

Nếu $n$ là số lẻ thì hoàn toàn thực hiện được. Tô tất cả số chẵn cùng màu đỏ, số lẻ cùng màu xanh. Khi đó, cách tô trên là thỏa vì:
(i) Mỗi số được tô bởi đúng 1 màu và có vô hạn lần tô mỗi màu (vì có vô hạn số lẻ, số chẵn)
(ii) Tổng $n$ số cùng màu là một số cùng màu vì tổng $n$ số lẻ là một số lẻ (vì $n$ lẻ) và tổng $n$ số chẵn là một số chẵn.
======================================
Nhưng nếu $n$ chẵn thì "có thể" không tô được. Nhưng không biết chứng minh sao :))

Cái trường hợp $n$ chẳn của bạn chỉ là điều kiện cần thôi :lol:
Bài này dù $n$ chẳn hay $n$ lẻ đều không tô được :)

Spoiler



#400613 Tô màu các số tự nhiên

Đã gửi bởi Stranger411 on 28-02-2013 - 09:13 trong Tổ hợp và rời rạc

Cho số tự nhiên $n>1$. Người ta tô màu tất cả các số tự nhiên bằng 2 màu xanh và đỏ thỏa mãn 2 đk:
1) Mỗi số chỉ được tô một màu và mỗi màu được dùng để tô vô hạn số.
2) Tổng của $n$ số phân biệt cùng màu, là một số cũng có màu đó.
Hỏi cách tô trên có thể thực hiện được hay không ?

Spoiler



#433417 Tìm số tự nhiên $n>4$ nhỏ nhất sa0 ch0 tồn tại $n$ ng...

Đã gửi bởi Stranger411 on 07-07-2013 - 00:26 trong Tổ hợp và rời rạc

Anh thử xem lại lời giải xem, người số 2 và 8 đâu thỏa mãn ạ. Đáp số $n=16$

khâu thử chọn anh làm sai @@!
đáp án đúng là $n=16$




#432485 Tìm số tự nhiên $n>4$ nhỏ nhất sa0 ch0 tồn tại $n$ ng...

Đã gửi bởi Stranger411 on 03-07-2013 - 11:14 trong Tổ hợp và rời rạc

Bài toán :

Tìm số tự nhiên $n>4$ nhỏ nhất sa0 ch0 tồn tại $n$ người thỏa mãn : 2 người quen nhau thì không có người quen chung còn 2 người không quen nhau thì có đúng 2 người quen chung !

Trước tiên ta chứng minh những người này có cùng người quen với nhau tại đây.
Cho mỗi người là một đỉnh của đồ thị. 2 người quen nhau thì được nối với nhau bằng 1 cạnh. Nên số người quen của mỗi người sẽ bằng với bậc của đỉnh. Suy ra đồ thì này có các đỉnh cùng bậc. Nói cách khác đây là một đồ thị đẳng cấu. Đồ thị này có $n$ đỉnh, các đỉnh có chung bậc $d$. Suy ra đồ thì có số cạnh là $m= \frac{nd}{2}$ với $d \ge 2$.

Ta tiếp tục chứng minh $|m| \leq 3(|n|-2)$.
Vì đồ thị này là liên thông nên theo định lí Euler suy ra: $|r|=|m|-|n|+2$ (r là số miền)
Gọi $n$ là tổng số cạnh thuộc các miền $r$ thì do mỗi cạnh chỉ thuộc nhiều nhất 2 miền nên $n \leq 2m$. Mỗi miền có ít nhất 3 cạnh nên $n \geq 3r$ từ đó suy ra $r \leq \frac{2}{3}m$. Thay vào công thức Euler ta đc đpcm.

Từ đó, ta có: $m= \frac{nd}{2} \le 3n-6$ $\rightarrow 12 \le (6-d)n$. Vậy $d \le 6$ (ta cần vế phải dương)

Theo các kết quả trên, ta thử chọn các trường hợp và chọn được $n_{min}=8$ với $d=3$.
Vậy $n_{min}=8$.




#347129 Tìm số nguyên $n> 1$ sao cho $\frac{2^{n...

Đã gửi bởi Stranger411 on 16-08-2012 - 09:52 trong Số học

BÀI TOÁN: Xác định tất cả các số nguyên $n> 1$ sao cho $\frac{2^{n}+1}{n^{2}}$ là một số nguyên.

Bài này ko cần phải dùng đến cấp của 1 số đâu :)

Bổ đề 1: Cho các số nguyên $m,n$ và $a>1$. Ta có: $\gcd \left( {{a^m} - 1,{a^n} - 1} \right) = {a^{\gcd \left( {m,n} \right)}} - 1$
Bổ đề 2: Nếu ${3^b}|{2^a} - 1 \Rightarrow {3^{b - 1}}|a$

Lời giải bài toán:
+ Khi $n=1$, bài toán thỏa mãn.
+ Khi $n>1 \Rightarrow n$ lẻ.
Gọi $p$ là ước nguyên tố lẻ nhỏ nhất của $n$ nên $\gcd \left( {p - 1,n} \right) = 1$.
Ta có: $p|{2^n} + 1|{2^{2n}} - 1$
Theo định lí Ferma nhỏ, ta có: $p|{2^{p - 1}} - 1$.
Áp dụng bổ đề 1, ta được: $p|\gcd \left( {{2^{p - 1}} - {{1,2}^{2n}} - 1} \right) = {2^{\gcd \left( {2n,p - 1} \right)}} - 1$
mà $\gcd \left( {2n,p - 1} \right) \leqslant 2 \Rightarrow p|3 \Rightarrow p = 3$
Đặt $n = {3^k}d$. Dùng bổ đề 2, ta có: ${3^{2k}}|{n^2}|{2^{2n}} - 1 \Rightarrow {3^{2k - 1}}|n \Rightarrow k = 1$
(*) Nếu $d>1$. Gọi $q$ là ước nguyên tố nhỏ nhất của $d$ nên $q \ge 5$.
Lập luận tương tự như trên, ta có: $q=7$
Vậy nên $7|n|{2^n} + 1$. Điều này vô lí vì ${2^n} + 1 \equiv 2,3,5(\bmod 7)$.
(*) Nếu $d=1$, ta có: $n=3$.
Vậy $n=1$ và $n=3$ thỏa mãn đề bài.



#347133 Tìm số nguyên $n> 1$ sao cho $\frac{2^{n...

Đã gửi bởi Stranger411 on 16-08-2012 - 10:06 trong Số học

Bài trên lâu lắm rồi :D Mình mở rộng bằng căn nguyên thủy tí nữa cho nó mạnh :D

Mở rộng: Tìm tất cả các số nguyên $n>1$ sao cho tồn tại duy nhất số nguyên $a$ với $0 < a < n!$ sao cho:
\[n!|{a^n} + 1\]

BÀI TOÁN: Xác định tất cả các số nguyên $n> 1$ sao cho $\frac{2^{n}+1}{n^{2}}$ là một số nguyên.

Bài này còn 2 cách giải nữa bằng căn nguyên thủy và LTE ;)



#409245 Tìm số dư của phép chia $S=\prod^{p}_{t=1}(t^...

Đã gửi bởi Stranger411 on 30-03-2013 - 22:03 trong Số học

Bài toán:

Ch0 số nguyên tố lẻ $p=mk+2$ tr0ng đó $m.k\in \mathbb{N}^{*},m>2$. Tìm số dư của phép chia $S=\prod^{p}_{t=1}(t^{m-1}+t^{m-2}+...+t+1)$ ch0 $p$.

Một bài khó đánh giá hơn ;)

Cho số nguyên tố $p \equiv 1( \bmod m)$, $m  >2$. Chứng minh:
\[\prod\limits_{t = 1}^p {\left( {{t^{m - 1}} + {t^{m - 2}} +  \ldots  + 1} \right)}  \equiv 0(\bmod p)\]




#359606 Tìm nghiệm nguyên

Đã gửi bởi Stranger411 on 06-10-2012 - 22:43 trong Số học

Tìm $p,q\in \mathbb{P}$ thỏa mãn $3pq\mid a^{3pq}-a$ với mọi $a\in \mathbb{Z}^+$

Nếu biết giới hạn $p,q$ và chọn $a$ là căn nguyên thủy của $n$ ngay từ đầu thì bài toán sẽ gọn hơn rất nhiều.
Giả sử $p \ge q$

+ Cho $a=3$, ta có:
${3^{pq}} \equiv 3\left( {\bmod 3pq} \right) \Rightarrow 3\left( {{3^{pq - 1}} - 1} \right) \vdots 3pq \Rightarrow p,q > 3$

+ Cho ${a^{\varphi \left( n \right)}} \equiv 1\left( {\bmod n} \right)$ ( $a$ là căn nguyên thủy của $n$)
Theo định lí Fermat nhỏ: ${a^{p - 1}} \equiv 1\left( {\bmod p} \right)$
Vì ${a^{3pq - 1}} \equiv 1\left( {\bmod p} \right) \Rightarrow p - 1|3pq - 1 \Rightarrow p - 1|3q - 1$
Chứng minh tương tự: $q - 1|3p - 1$
Vì $p \ge q$ nên $3q - 1 \in \left\{ {p - 1;2\left( {p - 1} \right);3\left( {p - 1} \right)} \right\}$
Thay vào điều kiện bài toán, ta được: $\boxed{(p,q)=(11,17),(17,11)}$



#346020 Tìm các số nguyên dương $a,b,c$ sao cho $\frac{a^{2}+b^{2...

Đã gửi bởi Stranger411 on 11-08-2012 - 23:47 trong Số học

He he áp dụng cái bổ đề anh Tường nói thì bài này làm ngon
Giải như sau:
Bổ đề: $p \in \mathbb{P}, p \equiv 2 \pmod{3}, a^2+3b^2 \vdots p \Leftrightarrow p|a,b$

Em Tạ giải kinh quá :P
Chắc thằng Tường nó chả bao giờ theo kịp đâu :P

Cách khác:
Đẳng thức được viết lại như sau $(a+b+c)^2=(3k+2)(ab+bc+ca)$
Chọn số nguyên tố $p$ sao cho $\left\{ \begin{gathered}
{p^{2a - 1}}|3k + 2 \\
{p^{2a}}|3k + 2 \\
\end{gathered} \right.$
$ \Rightarrow \left\{ \begin{gathered}
{p^a}|a + b + c \\
p|ab + bc + ca \\
\end{gathered} \right.$
$c \equiv - a - b(\bmod p) \Rightarrow p|a^2 + ab + b^2 \Rightarrow p|{(2a + b)^2} + 3{b^2}$
$ \Rightarrow \left( {\frac{{ - 3}}{p}} \right) = 1$. Và điều này vô lí vì $p \equiv 2(\bmod 3)$.
Vậy không tồn tại $a,b,c$ thỏa mãn bài toán. $\blacksquare$



#347148 Tìm các số nguyên dương $a,b,c$ sao cho $\frac{a^{2}+b^{2...

Đã gửi bởi Stranger411 on 16-08-2012 - 10:37 trong Số học

sai từ chỗ này và nguyên nhân là do làm tắt $p|{(2a + b)^2} + 3{b^2}$
$ \Rightarrow \left( {\frac{{ - 3}}{p}} \right) = 1$
muốn dùng lengdre(hay tiếng việ gọi là thặng dư toàn phương) trước tiên ta phải đưa nó về dạng (mà ở đây) là
a2$\equiv$-3 (mod p) cái đã,mà ở đây muốn đưa về dạng này ta phải giả sử a không chia hết cho p,''vậy nên thiếu TH a,b chia hết cho p'',mà TH này luôn đúng,nếu không thấy dc thì cho a=b=p ta có 12p2 chia hết cho p ,vì vậy có giải kiểu gì đi nữa vẫn phải thông qua a,b,c chia hết cho p rồi mới giải tiếp,nên không có cách bạn stranger nói

Nói chuyện vs Uyenha cực kì bực mình @@!
Mình và mọi người đã ko muốn nói rồi mà bạn cứ thích cãi cùn.

Trước đó, MOD đã gộp bớt vài bài của bạn để tránh spam trong topic.
Thắc mắc thì ko phải là tội nhưng cứ nói dai như thế người ta chả thích tí nào đâu bạn :)

Mời bạn tham khảo thêm về kí hiệu Lengdre:
File gửi kèm  Cong Thuc Legendre.pdf   67.83K   1680 Số lần tải



#346426 Tìm các số nguyên dương $a,b,c$ sao cho $\frac{a^{2}+b^{2...

Đã gửi bởi Stranger411 on 13-08-2012 - 12:03 trong Số học

ý m` là không có cách của bạn đâu,nếu 2a+b chia hết cho p thì làm quoái j` mà vô lí chứ,cũng giống như a mũ 2 + b mũ 2 có ước nguyên tố dạng thì 4p+3 thì a,b cùng chia hết cho SNT đó,làm j` có chuyện vô lí ở đâu,khi sử dụng kí hiêu lengdre ta đã ngầm hiểu tử số của nó không chia hết cho mẫu r` :lol:

Trước khi nói cái gì thì nên coi lại kiến thức của mình một tí đi nhá ;)

Định nghĩa về kí hiệu Lengdre:
Cho số nguyên tố $p$ và số nguyên $a$. Khi đó, ta có:
+$\left( {\frac{{ a}}{p}} \right) = 1$ nếu $a \not \vdots p$ và $a$ là số chính phương $mod(p)$
+$\left( {\frac{{ a}}{p}} \right) = -1$ nếu $a \not \vdots p$ và $a$ không là số chính phương $mod(p)$
+$\left( {\frac{{ a}}{p}} \right) = 0$ nếu $a \vdots p$

Trờ lại bài toán:
Vì vậy nếu $\left( {\frac{{ - 3}}{p}} \right) = 1$ thì hoàn toàn vô lí vì ta chọn $p \equiv 2(\bmod 3)$



#346577 Tìm các số nguyên dương $a,b,c$ sao cho $\frac{a^{2}+b^{2...

Đã gửi bởi Stranger411 on 13-08-2012 - 21:29 trong Số học

$\Rightarrow 2a^2+2ab+2b^2 \vdots p \Rightarrow 4a^2+4ab+4b^2 \vdots p \Rightarrow (2a+b)^2+3b^2 \vdots p$

$c \equiv - a - b(\bmod p) \Rightarrow p|a^2 + ab + b^2 \Rightarrow p|{(2a + b)^2} + 3{b^2}$
$ \Rightarrow \left( {\frac{{ - 3}}{p}} \right) = 1$. Và điều này vô lí vì $p \equiv 2(\bmod 3)$.
Vậy không tồn tại $a,b,c$ thỏa mãn bài toán. $\blacksquare$

Bạn xem lại @@!
Bài mình và bài Tạ để đi đến $p|{(2a + b)^2} + 3{b^2}$ để suy ra vô lí mà (:|

Bạn nói lại xem mình sai chỗ nào (:|



#346392 Tìm các số nguyên dương $a,b,c$ sao cho $\frac{a^{2}+b^{2...

Đã gửi bởi Stranger411 on 13-08-2012 - 09:49 trong Số học

muốn có -3 là thặng dư toàn phương của p thì 1 trong 2 số 2a+b hoặc b không chia hết cho p,nên theo nguyenta a,b,c chia hết cho p là hợp lí

Gì nữa đây bạn :P
$\left( {\frac{{ - 3}}{p}} \right) = 1$ là kí hiệu Lengdre ;) Chớ có phải thăng dư toàn phuơng gì đâu ;))
Cách em Nguyenta98 là lùi vô hạn. Cách mình là dùng các định lí về thăng dư bậc 2 thôi ;)



#312453 Topic: INEQUALITIES (PART II)

Đã gửi bởi Stranger411 on 24-04-2012 - 19:32 trong Bất đẳng thức - Cực trị

Problem 8: Cho a,b,c là các số thực thỏa $a+b+c=3$. Chứng minh rằng:
$$\frac{a^2-bc}{a^2+3}+\frac{b^2-ac}{b^2+3}+\frac{c^2-ab}{c^2+3}\geq 0$$

Bài này khá yếu.
Bằng cách phân tích trực tiếp, ta được:
$$\left( {{a}^{2}}+{{b}^{2}}+{{c}^{2}}-ab-bc-ca \right)\sum{\frac{{{\left( a-b \right)}^{2}}}{\left( {{a}^{2}}+3 \right)\left( {{b}^{2}}+3 \right)}}\ge 0$$
Ta có đpcm. $\blacksquare$

Problem 9: Cho a,b,c là các số thực thỏa $a+b+c=1$. Chứng minh rằng:
$$\frac{\left( {{a}^{2}}+{{b}^{2}} \right)}{{{\left( a+b \right)}^{2}}}\frac{\left( {{c}^{2}}+{{b}^{2}} \right)}{{{\left( c+b \right)}^{2}}}\frac{\left( {{a}^{2}}+{{c}^{2}} \right)}{{{\left( a+c \right)}^{2}}}\ge \frac{3}{8}\left( {{a}^{2}}+{{b}^{2}}+{{c}^{2}} \right)$$



#432263 Topic về tổ hợp, các bài toán về tổ hợp

Đã gửi bởi Stranger411 on 02-07-2013 - 12:05 trong Tổ hợp và rời rạc

Post tiếp 2 bài nữa làm ... điểm tâm  :wacko: :

Bài 15:(APMO 1998) Cho F là tập tất cả các bộ (A1, . . . ,An) sao cho mỗi Ai là một tập con của {1, 2, . . . , 1998}. Ký hiệu |A| là số các phần tử của tập hợp A. Hãy tính

$\sum\limits_{({A_1},{A_2},...,{A_n}) \in F} {|{A_1} \cup {A_2} \cup ... \cup {A_n}|} $

Bài 16:(Bulgaria 1996) HÌnh chữ nhật $m \times n ( m,n \in N, m,n>1)$ được chia thành $mn$ hình vuông đơn vị. Có bao nhiêu cách xoá 2 hình vuông sao cho phần còn lại có thể lát kín bởi các domino?

Tại sao các bạn cứ thích post những bài không có lời giải ở trong sách ra thế nhỉ :))
Mình nhớ không nhầm thì mấy bài này trong chuyên đề của thầy Huỳnh Tấn Châu để học sinh tự giải. Sau đây là cách của mình, bạn có cách khác thì post lên cho mọi người tham khảo.
Giải bài 15:

Bài này đếm bằng truy hồi.
Tập $(1, 2, . . . , 1998)$ có tất cả $2^{1998}$ tập con $A_i$ nên tổng cần tính có tất cả $2^{1998n}$ bộ $n$ số.
Bước 1: Với các tập con $(A_1, A_2 ... , A_n)$ của $(1, 2, . . . , 1997)$, ta có thể thêm hoặc không thêm vào mỗi tập $A_i$ phần tử $1998$ để tạo thành tập con của tập $(1, 2, . . . , 1998)$. Vậy từ bộ $(A_1, A_2 ... , A_n)$ ta có $2^n$ bộ gồm các tập con của $\left \{1, 2, . . . , 1998 \right \}$.

Bước 2: Bây h chỉ còn xét các tập con $(A_1, A_2 ... , A_n)$ của các bộ còn lại. Làm tương tự như trên thì ta có được $(2^n -1).2^{n(m-1)}$.
Tình tổng lại, ta có:

$\sum\limits_{({A_1},{A_2},...,{A_n}) \in (1,...,1998)} {|{A_1} \cup {A_2} \cup ... \cup {A_n}|} = (2^n.\sum\limits_{({A_1},{A_2},...,{A_n}) \in (1,...,1998)} {|{A_1} \cup {A_2} \cup ... \cup {A_n}|}) + (2^n -1) 2^{n(m-1)}$
Từ đó, ta có: $\sum\limits_{({A_1},{A_2},...,{A_n}) \in F} {|{A_1} \cup {A_2} \cup ... \cup {A_n}|} = 1998(2^{1998n} - 2^{1997n})$.

 

Mở rộng bài 15: Tính:
$P= \sum_{(A_{1},A_{2},...,A_{k})\in F_{k}}\sum_{b\in (A_{1}\cup A_{2}\cup ...\cup A_{k})}b$
$S= \sum_{(A_{1},A_{2},...,A_{k})\in F_{k}}\sum_{i=1}^{k}\sum_{b\in A_{i}}b$
 

Bài 7: (Đề đề nghị 30/4 -2012) Trong một kì thi hoa hậu, mỗi thành viên của ban giám khảo được quyền đề xuất $10$ người đẹp vào vòng chung kết. Một nhóm người đẹp được gọi là nhóm ưng ý đối với giám khảo A nếu trong nhóm đó có ít nhất một người đẹp mà A đề xuất. Biết rằng với 6 giám khảo bất kỳ luôn tồn tại một nhóm gồm 2 người đẹp là nhóm ưng ý đối với mỗi giám khảo trong 6 giám khảo đó. Chứng minh rằng tồn tại một nhóm gồm 10 người đẹp là nhóm ưng ý đối với mỗi thành viên của ban giám khảo.

Đề này của trường chuyên Nguyễn Tất Thành - Komtum và họ không đưa đáp án nên trong sách không có lời giải.
Mình nghĩ bạn Ispectorgadget cũng không có đáp án cho bài này. (Trừ khi bạn ấy học trường chuyên NTT)

Bài này chắc không thể giải bằng đồ thị vì phải xét tới 2 đối tượng là hoa hậu và giám khảo.
Thôi thì phát biểu lại dưới dạng tập hợp để mọi người cùng nghiên cứu:
Cho $X=\{1;2;...;n\}$. Và $A_1 ,A_2, ..., A_m$ là các tập con của $X$.
Biết 6 phần tử bất kì của $X$ luôn thuộc $|A_i\cup A_j|$ nào đó.
Chứng minh tồn tại $i_1;i_2;..;i_{10}$ phân biệt mà $|\bigcup\limits_{j=1}^{10}A_{i_j}|=X$.




#431535 Topic về tổ hợp, các bài toán về tổ hợp

Đã gửi bởi Stranger411 on 29-06-2013 - 11:02 trong Tổ hợp và rời rạc

Bài 2: (PP ánh xạ, chứng minh đặc tính tổ hợp) Gọi an là số các xâu nhị phân độ dài n không chứa chuỗi con 010, bn là số các xâu nhị phân độ dài n không chứa chuỗi con 0011 hoặc 1100. Chứng minh rằng bn+1 = 2an với mọi n nguyên dương. 

Giải bài 2:
Xét một xâu nhị phân bất kì  $\left \{ x_1, x_2,...,x_n \right \}$.
Ta xây dựng một xâu nhị phân $\left \{ y_1, y_2,...,y_{n+1} \right \}$ sao cho $y_1 =0$ và $y_k = x_1 + x_2 +...+ x_k(mod2)$
Rõ ràng, xâu $\left \{ x_1, x_2,...,x_n \right \}$ có dạng $a_n$ khi và chỉ khi $\left \{ y_1, y_2,...,y_{n+1} \right \}$ có dạng $b_n$. Nói cách khác đó là một song ánh biến các xâu có dạng $a_n$ thành các xâu có dạng $b_{n+1}$ bắt đầu bằng $0$.
Với mỗi xâu $\left \{ y_1, y_2,...,y_{n+1} \right \}$, ta thay các kí tự 1 bởi 0 và 0 bởi 1, ta được một xâu khác cũng có dạng $b_{n+1}$ nhưng bắt đầu bởi $1$.
Từ đó cho ta: $b_{n+1} = 2a_n$.
 

Bài 3 (Toán trò chơi-Nguồn: Tournament of the Towns). Trên bảng ô vuông $20$x$20$ mỗi ô có $1$ quân cờ. $A$ chọn một số thực $d$ và $B$ phải đưa mỗi quân cờ đi tới ô cách ô ban đầu nó đứng một khoảng cách ít nhất là $d$ (khoảng cách giữa $2$ ô được tính theo khoảng cách tâm của $2$ ô đó) và mỗi ô chỉ có thể có $1$ quân cờ. Hỏi với điều kiện gì của $d$ thì $B$ có thể thao tác thoả mãn điều kiện trên?

Hỏi tương tự với bảng $21$x$21$

Với bài này có lẽ ta phải chia bàn cờ ra 4 phần.
Cái đáp số hơi lằng nhằng, không biết có đúng không.




#432471 Topic về tổ hợp, các bài toán về tổ hợp

Đã gửi bởi Stranger411 on 03-07-2013 - 10:00 trong Tổ hợp và rời rạc

Bài 17 (cực trị-bất đẳng thức tổ hợp)

Cho tập hợp $X=\left \{ 1,2,...,50 \right \}$. Tìm số nguyên dương nhỏ nhất $k$ sao cho mọi tập con gồm $k$ phần tử của $X$ đều chứa hai phần tử phân biệt $a$ và $b$ sao cho $ab$ chia hết cho $a+b$

Ta có các cặp số thoả mãn $ab$ chia hết $a+b$: $(3;6);(4;12);(5;20);(6;12);(6;30);(7;42);(8;2);(9;18);(10;15);(10;40);(12;24);(12;36);(14;35);(15;30);(16;48);(18;36);(20;30);(21;28);(21;42);(24;40);(24;48);(30;45);(36;45)$

Tổng hợp các số lại tập $A$, ta có:\[A = \left\{ {3;4;5;6;7;8;9;10;12;14;15;16;18;20;21;24;30;35;36;40;45;48} \right\}\]

\[ \Rightarrow k \ge \left| {X\backslash A} \right| + \left[ {\frac{{\left| A \right|}}{2}} \right] + 1 = 39\]

Vậy $min(k)=39$

--------------------------------------------------------------------------------------------------------------------------------

Đây là kết quả sau 1h bấm máy tính của mình, các bạn có cách nào hay hơn thì đưa lên để mọi người cùng tham khảo

Giải bài 17:
Chứng minh $k \ge 39$
Các cặp số thoả mãn $ab$ chia hết $a+b$: $(3;6);(4;12);(5;20);(6;12);(6;30);(7;42);(8;2);(9;18);(10;15);(10;40);(12;24);(12;36);(14;35);(15;30);(16;48);(18;36);(20;30);(21;28);(21;42);(24;40);(24;48);(30;45);(36;45)$

Xét tập $M=(6,12,15,18,20,21,24,35,40,42,45,48)$. Vì 23 cặp trên đều có phần tử thuộc $M$ nên tập $X\M$ không thỏa mãn bài toán. Mà $|X\M|=38 \rightarrow k \ge 39$

Chứng minh mọi tập có 39 phần tử đều thỏa mãn bài toán:
Xét tập $A$ bất kì gồm 39 phần tử của $X$.
Chọn 12 cặp số trong 23 cặp trên sao cho các phần tử không trùng nhau: $(3;6);(4;12);(5;20);(7;42);(8;2);(9;18);(10;15);(14;35);(18;36);(21;28);(24;40);(30;45)$

12 cặp trên chứa 24 phần tử của $X$. Nên $X$ chỉ còn lại 26 phần tử.
Vậy ít nhất 13 phần tử của $A$ của phải thuộc 12 cặp trên.
Theo nguyên lí drichlet thì $A$ chứa ít nhất 1 trong 12 cặp trên.
Từ đó tập $A$ thỏa mãn đề bài.




#431947 Topic về tổ hợp, các bài toán về tổ hợp

Đã gửi bởi Stranger411 on 30-06-2013 - 22:38 trong Tổ hợp và rời rạc

Bài 11:(Lý thuyết đồ thị, APMO 1989) Chứng minh rằng một đồ thị n đỉnh, k cạnh thì sẽ có ít nhất $\frac{{k\left( {4k - {n^2}} \right)}}{{3n}}$ tam giác.

Câu này được chế biến lại làm đề chọn đội tuyển của KHTN năm nào đó thì phải, nhưng cũng chỉ là điều kiện cần.
Giải bài 11:
Gọi $D_i$ là bậc của đỉnh $i$ nào đó.
Nếu 2 đỉnh $i$ và $j$ liên thông thì tồn tại ít nhất $D_i + D_j - 2$ cạnh khác nổi đến $n-2$ đỉnh khác của đồ thị.
Nên có $D_i + D_j - n$ tam giác có đỉnh là $j$ và $j$. Vậy số tam giác có chứa cạnh $ij$ được tạo thành phải tối thiểu là:
$\sum_{i,j} {\frac{D_i + D_j -n}{3}}=\sum_{i,j} {\frac{D_i + D_j}{3} - \frac{nk}{3}} = \sum_{i} {\frac{D_i^2}{3} - \frac{nk}{3}}$

Vậy số tam giác tối thiểu được tạo thành là:

$\sum_{i}{\frac{D_i^2}{3}} -\frac{nk}{3} \geq \frac{1}{3n}\left ( \sum_{i}{D_i} \right )^2 - \frac{nk}{3} = \frac{4k^2}{3n} -\frac{nk}{3}$
Q.E.D.

 

Bài 6: (truy hồi/đa thức,giải tích tổ hợp,PTNK 2009) Cho số nguyên dương n. Có bao nhiêu số chia hết cho 3, có n chữ số và các chữ số đều thuộc {3, 4, 5, 6}?

Sau đây là cách giải PP truy hồi:

Gọi $a_n$ là số các số có $n$ chữ số lập từ ${3, 4, 5, 6}$ và chia hết cho 3, còn $b_n$ là số các số có $n$ chữ số lập từ ${3, 4, 5, 6}$ và không chia hết cho 3. Khi đó ta có

$a_n = 2a_{n-1} + b_{n-1}(1)$

$b_n = 2a_{n-1} + 3b_{n-1}(2)$

Từ (1) suy ra $b_{n-1} = a_n – 2a_{n-1}$, thay n à n+1 thì được $b_n = a_{n+1} – 2a_n$. Thay vào (2), ta được

$a_{n+1} – 2a_n = 2a_{n-1} + 3(a_n – 2a_{n-1})$

$a_{n+1} – 5a_n + 4a_{n-1} = 0$.

Giải phương trình sai phân này, với chú ý rằng a1 = 2, a2 = 6, ta tìm được

\[{a_n} = \frac{{{4^n} + 2}}{3}.\]

Với cách đặt trên ta có thể có công thức truy hồi khác là $b_n = 4^n - a_n$
từ đó suy ra $a_{n+1} = 2a_{n} + b_{n}= a_n + 4^n$
Nên $a_{n} = 4^{n-1} + 4^{n-2} +...+ a_{1} = \frac{4^n + 2}{3}$




#372273 Phân hoạch tập tốt

Đã gửi bởi Stranger411 on 24-11-2012 - 23:13 trong Tổ hợp và rời rạc

Cho số nguyên dương $m \ge 3$. Một tập $S$ được gọi là tốt nếu tồn tại những phần tử ${s_1},{s_2},...,{s_m}$ thỏa mãn ${s_1}+{s_2}+...+{s_{m-1}}={s_m}$. Xác định số nguyên dương $f(m)$ nhỏ nhất sao cho một trong 2 phân hoạch A và B của tập ${ 1,2,...,f(m) }$ là 1 tập tốt.



#398293 Những người phát cuồng vì tramyvodoi

Đã gửi bởi Stranger411 on 19-02-2013 - 19:01 trong Góc giao lưu

Post cái hình lên :v
Bạn nào thích thì cứ lấy làm ava lun nhá :-j

Hình đã gửi



#397889 Những người phát cuồng vì tramyvodoi

Đã gửi bởi Stranger411 on 17-02-2013 - 23:58 trong Góc giao lưu

Các cháu trên faceboook đang phát cuồng :D
Mong bạn tramyvodoi đừng giận nhá :))
Hình đã gửi

Và đây là hậu quả của những cháu đua đòi :-ss
Hình đã gửi



#441245 Mở rộng Problem 4 - IMO 2013

Đã gửi bởi Stranger411 on 08-08-2013 - 14:40 trong Hình học

1 bài mở rộng của thầy Quang Hùng trong GGTH lần 5

Cho tam giác $ABC$, đường tròn $(I)$ đi qua $B,C$ cắt CA,AB tại $N,M$ khác $B,C$.
Đặt $H = BN \cap CM$. Gọi $d$ là đường thẳng qua $I$ vuông góc với $AH$. Lấy $W$ bất kì trên $d$.
$WK,WL$ là đường kính của đường tròn ngoại tiếp các tam giác $WBM,WCN$.
Chứng minh $K,H,L$ thẳng hàng.
 

TBr.png




#343075 MOSP 2001 by Cecil Rousseseau

Đã gửi bởi Stranger411 on 03-08-2012 - 13:15 trong Tổ hợp và rời rạc

Problem: $a_n$ kí hiệu là số tập con không rỗng của $S$ thỏa mãn rằng:
(i) $S\subseteq${$1$, $2$, $...$, $n$};
(ii) tất cả các phần tử của $S$ đều cung tính chẵn, lẻ.
(iii) mỗi phân tử $k\in{S}$ thỏa mãn $k\geq2|S|$.
Tìm công thức tường minh cho $a_n$

Bài này lâu rồi, sử dụng phép chia nhóm là được :)

Ta có: $ a_{2m-1}= 2(F_{m+1}-1) $ và $ a_{2m}= F_{m+3}-2 $
với $m\ge1$ và $F_{m}$ là số Fibonacci thứ $m$.


Lời giải:
Đặt $T_{n}=\{S\in\{1,2,\cdots,n\}\}$ thỏa $(ii)$ và $(iii)$
Chia $ T_{n+4} $ thành 3 tập con:

Phần 1: $A_{n+4}=\{S\in T_{n+4}\ ;\ 1,2\notin S,\ \forall k\in S, k\geq 2|S|+2\}$
Xây dựng $ f\ :\ \mathcal{P}(\{1,2,\cdots,n+2\})\rightarrow\mathcal{P}(\{1,2,\cdots,n+4\}) $ thỏa:
$$f(\{x_{1},x_{2},\cdots,x_{k}\}) =\{x_{1}+2,x_{2}+2,\cdots,x_{k}+2\}$$
Ta được: $ f(T_{n+2}) = A_{n+4} $ nên $ |A_{n+4}|=|T_{n+2}|$

Phần 2: $ B_{n+4}=\{S\in T_{n+4}\ ;\ 1,2\notin S,\ \exists k\in S, k < 2|S|+2\} $
Tương tự, ta được:

$f(\phi) =\{3\}$
$f\left( {\left\{ {{x_1},{x_2},...,{x_k}} \right\}} \right) = \left\{ {{x_1} + 4,{x_2} + 4,...,{x_k} + 4,2k} \right\}$
nếu các phần tử cùng chẳn.
$f\left( {\left\{ {{x_1},{x_2},...,{x_k}} \right\}} \right) = \left\{ {{x_1} + 4,{x_2} + 4,...,{x_k} + 4,2k+1} \right\}$
nếu các phần tử cùng lẽ.
Suy ra: $|B_{n+4}|=|T_{n}|+1 $

Phần 3: $ C_{n+4}=\{ S\in T_{n+4}\ ;\ 1\in S\ \mathrm{or}\ 2\in S\} $
Tương tự, ta có: $ |T_{n+4}|=|T_{n+2}|+|T_{n}|+2 $

Từ đó, ta chứng minh được: $ a_{2m-1}= 2(F_{m+1}-1) $ và $ a_{2m}= F_{m+3}-2 $



#441489 IMO shortlist 2012 - Problems + Solution

Đã gửi bởi Stranger411 on 09-08-2013 - 15:20 trong Thi HSG Quốc gia và Quốc tế

Mới có bản Scan thôi, các bạn dùng tạm :)

File gửi kèm